Javatpoint Logo

Machine Learning

Artificial Intelligence

Control System

Supervised Learning

Classification, miscellaneous, related tutorials.

Interview Questions

JavaTpoint

The hypothesis is a common term in Machine Learning and data science projects. As we know, machine learning is one of the most powerful technologies across the world, which helps us to predict results based on past experiences. Moreover, data scientists and ML professionals conduct experiments that aim to solve a problem. These ML professionals and data scientists make an initial assumption for the solution of the problem.

This assumption in Machine learning is known as Hypothesis. In Machine Learning, at various times, Hypothesis and Model are used interchangeably. However, a Hypothesis is an assumption made by scientists, whereas a model is a mathematical representation that is used to test the hypothesis. In this topic, "Hypothesis in Machine Learning," we will discuss a few important concepts related to a hypothesis in machine learning and their importance. So, let's start with a quick introduction to Hypothesis.

It is just a guess based on some known facts but has not yet been proven. A good hypothesis is testable, which results in either true or false.

: Let's understand the hypothesis with a common example. Some scientist claims that ultraviolet (UV) light can damage the eyes then it may also cause blindness.

In this example, a scientist just claims that UV rays are harmful to the eyes, but we assume they may cause blindness. However, it may or may not be possible. Hence, these types of assumptions are called a hypothesis.

The hypothesis is one of the commonly used concepts of statistics in Machine Learning. It is specifically used in Supervised Machine learning, where an ML model learns a function that best maps the input to corresponding outputs with the help of an available dataset.

There are some common methods given to find out the possible hypothesis from the Hypothesis space, where hypothesis space is represented by and hypothesis by Th ese are defined as follows:

It is used by supervised machine learning algorithms to determine the best possible hypothesis to describe the target function or best maps input to output.

It is often constrained by choice of the framing of the problem, the choice of model, and the choice of model configuration.

. It is primarily based on data as well as bias and restrictions applied to data.

Hence hypothesis (h) can be concluded as a single hypothesis that maps input to proper output and can be evaluated as well as used to make predictions.

The hypothesis (h) can be formulated in machine learning as follows:

Where,

Y: Range

m: Slope of the line which divided test data or changes in y divided by change in x.

x: domain

c: intercept (constant)

: Let's understand the hypothesis (h) and hypothesis space (H) with a two-dimensional coordinate plane showing the distribution of data as follows:

Hypothesis space (H) is the composition of all legal best possible ways to divide the coordinate plane so that it best maps input to proper output.

Further, each individual best possible way is called a hypothesis (h). Hence, the hypothesis and hypothesis space would be like this:

Similar to the hypothesis in machine learning, it is also considered an assumption of the output. However, it is falsifiable, which means it can be failed in the presence of sufficient evidence.

Unlike machine learning, we cannot accept any hypothesis in statistics because it is just an imaginary result and based on probability. Before start working on an experiment, we must be aware of two important types of hypotheses as follows:

A null hypothesis is a type of statistical hypothesis which tells that there is no statistically significant effect exists in the given set of observations. It is also known as conjecture and is used in quantitative analysis to test theories about markets, investment, and finance to decide whether an idea is true or false. An alternative hypothesis is a direct contradiction of the null hypothesis, which means if one of the two hypotheses is true, then the other must be false. In other words, an alternative hypothesis is a type of statistical hypothesis which tells that there is some significant effect that exists in the given set of observations.

The significance level is the primary thing that must be set before starting an experiment. It is useful to define the tolerance of error and the level at which effect can be considered significantly. During the testing process in an experiment, a 95% significance level is accepted, and the remaining 5% can be neglected. The significance level also tells the critical or threshold value. For e.g., in an experiment, if the significance level is set to 98%, then the critical value is 0.02%.

The p-value in statistics is defined as the evidence against a null hypothesis. In other words, P-value is the probability that a random chance generated the data or something else that is equal or rarer under the null hypothesis condition.

If the p-value is smaller, the evidence will be stronger, and vice-versa which means the null hypothesis can be rejected in testing. It is always represented in a decimal form, such as 0.035.

Whenever a statistical test is carried out on the population and sample to find out P-value, then it always depends upon the critical value. If the p-value is less than the critical value, then it shows the effect is significant, and the null hypothesis can be rejected. Further, if it is higher than the critical value, it shows that there is no significant effect and hence fails to reject the Null Hypothesis.

In the series of mapping instances of inputs to outputs in supervised machine learning, the hypothesis is a very useful concept that helps to approximate a target function in machine learning. It is available in all analytics domains and is also considered one of the important factors to check whether a change should be introduced or not. It covers the entire training data sets to efficiency as well as the performance of the models.

Hence, in this topic, we have covered various important concepts related to the hypothesis in machine learning and statistics and some important parameters such as p-value, significance level, etc., to understand hypothesis concepts in a better way.





Youtube

  • Send your Feedback to [email protected]

Help Others, Please Share

facebook

Learn Latest Tutorials

Splunk tutorial

Transact-SQL

Tumblr tutorial

Reinforcement Learning

R Programming tutorial

R Programming

RxJS tutorial

React Native

Python Design Patterns

Python Design Patterns

Python Pillow tutorial

Python Pillow

Python Turtle tutorial

Python Turtle

Keras tutorial

Preparation

Aptitude

Verbal Ability

Interview Questions

Company Questions

Trending Technologies

Artificial Intelligence

Cloud Computing

Hadoop tutorial

Data Science

Angular 7 Tutorial

B.Tech / MCA

DBMS tutorial

Data Structures

DAA tutorial

Operating System

Computer Network tutorial

Computer Network

Compiler Design tutorial

Compiler Design

Computer Organization and Architecture

Computer Organization

Discrete Mathematics Tutorial

Discrete Mathematics

Ethical Hacking

Ethical Hacking

Computer Graphics Tutorial

Computer Graphics

Software Engineering

Software Engineering

html tutorial

Web Technology

Cyber Security tutorial

Cyber Security

Automata Tutorial

C Programming

C++ tutorial

Data Mining

Data Warehouse Tutorial

Data Warehouse

RSS Feed

MLP Logo

Hypothesis Testing – A Deep Dive into Hypothesis Testing, The Backbone of Statistical Inference

  • September 21, 2023

Explore the intricacies of hypothesis testing, a cornerstone of statistical analysis. Dive into methods, interpretations, and applications for making data-driven decisions.

use of hypothesis in machine learning

In this Blog post we will learn:

  • What is Hypothesis Testing?
  • Steps in Hypothesis Testing 2.1. Set up Hypotheses: Null and Alternative 2.2. Choose a Significance Level (α) 2.3. Calculate a test statistic and P-Value 2.4. Make a Decision
  • Example : Testing a new drug.
  • Example in python

1. What is Hypothesis Testing?

In simple terms, hypothesis testing is a method used to make decisions or inferences about population parameters based on sample data. Imagine being handed a dice and asked if it’s biased. By rolling it a few times and analyzing the outcomes, you’d be engaging in the essence of hypothesis testing.

Think of hypothesis testing as the scientific method of the statistics world. Suppose you hear claims like “This new drug works wonders!” or “Our new website design boosts sales.” How do you know if these statements hold water? Enter hypothesis testing.

2. Steps in Hypothesis Testing

  • Set up Hypotheses : Begin with a null hypothesis (H0) and an alternative hypothesis (Ha).
  • Choose a Significance Level (α) : Typically 0.05, this is the probability of rejecting the null hypothesis when it’s actually true. Think of it as the chance of accusing an innocent person.
  • Calculate Test statistic and P-Value : Gather evidence (data) and calculate a test statistic.
  • p-value : This is the probability of observing the data, given that the null hypothesis is true. A small p-value (typically ≤ 0.05) suggests the data is inconsistent with the null hypothesis.
  • Decision Rule : If the p-value is less than or equal to α, you reject the null hypothesis in favor of the alternative.

2.1. Set up Hypotheses: Null and Alternative

Before diving into testing, we must formulate hypotheses. The null hypothesis (H0) represents the default assumption, while the alternative hypothesis (H1) challenges it.

For instance, in drug testing, H0 : “The new drug is no better than the existing one,” H1 : “The new drug is superior .”

2.2. Choose a Significance Level (α)

When You collect and analyze data to test H0 and H1 hypotheses. Based on your analysis, you decide whether to reject the null hypothesis in favor of the alternative, or fail to reject / Accept the null hypothesis.

The significance level, often denoted by $α$, represents the probability of rejecting the null hypothesis when it is actually true.

In other words, it’s the risk you’re willing to take of making a Type I error (false positive).

Type I Error (False Positive) :

  • Symbolized by the Greek letter alpha (α).
  • Occurs when you incorrectly reject a true null hypothesis . In other words, you conclude that there is an effect or difference when, in reality, there isn’t.
  • The probability of making a Type I error is denoted by the significance level of a test. Commonly, tests are conducted at the 0.05 significance level , which means there’s a 5% chance of making a Type I error .
  • Commonly used significance levels are 0.01, 0.05, and 0.10, but the choice depends on the context of the study and the level of risk one is willing to accept.

Example : If a drug is not effective (truth), but a clinical trial incorrectly concludes that it is effective (based on the sample data), then a Type I error has occurred.

Type II Error (False Negative) :

  • Symbolized by the Greek letter beta (β).
  • Occurs when you accept a false null hypothesis . This means you conclude there is no effect or difference when, in reality, there is.
  • The probability of making a Type II error is denoted by β. The power of a test (1 – β) represents the probability of correctly rejecting a false null hypothesis.

Example : If a drug is effective (truth), but a clinical trial incorrectly concludes that it is not effective (based on the sample data), then a Type II error has occurred.

Balancing the Errors :

use of hypothesis in machine learning

In practice, there’s a trade-off between Type I and Type II errors. Reducing the risk of one typically increases the risk of the other. For example, if you want to decrease the probability of a Type I error (by setting a lower significance level), you might increase the probability of a Type II error unless you compensate by collecting more data or making other adjustments.

It’s essential to understand the consequences of both types of errors in any given context. In some situations, a Type I error might be more severe, while in others, a Type II error might be of greater concern. This understanding guides researchers in designing their experiments and choosing appropriate significance levels.

2.3. Calculate a test statistic and P-Value

Test statistic : A test statistic is a single number that helps us understand how far our sample data is from what we’d expect under a null hypothesis (a basic assumption we’re trying to test against). Generally, the larger the test statistic, the more evidence we have against our null hypothesis. It helps us decide whether the differences we observe in our data are due to random chance or if there’s an actual effect.

P-value : The P-value tells us how likely we would get our observed results (or something more extreme) if the null hypothesis were true. It’s a value between 0 and 1. – A smaller P-value (typically below 0.05) means that the observation is rare under the null hypothesis, so we might reject the null hypothesis. – A larger P-value suggests that what we observed could easily happen by random chance, so we might not reject the null hypothesis.

2.4. Make a Decision

Relationship between $α$ and P-Value

When conducting a hypothesis test:

  • We first choose a significance level ($α$), which sets a threshold for making decisions.

We then calculate the p-value from our sample data and the test statistic.

Finally, we compare the p-value to our chosen $α$:

  • If $p−value≤α$: We reject the null hypothesis in favor of the alternative hypothesis. The result is said to be statistically significant.
  • If $p−value>α$: We fail to reject the null hypothesis. There isn’t enough statistical evidence to support the alternative hypothesis.

3. Example : Testing a new drug.

Imagine we are investigating whether a new drug is effective at treating headaches faster than drug B.

Setting Up the Experiment : You gather 100 people who suffer from headaches. Half of them (50 people) are given the new drug (let’s call this the ‘Drug Group’), and the other half are given a sugar pill, which doesn’t contain any medication.

  • Set up Hypotheses : Before starting, you make a prediction:
  • Null Hypothesis (H0): The new drug has no effect. Any difference in healing time between the two groups is just due to random chance.
  • Alternative Hypothesis (H1): The new drug does have an effect. The difference in healing time between the two groups is significant and not just by chance.
  • Choose a Significance Level (α) : Typically 0.05, this is the probability of rejecting the null hypothesis when it’s actually true

Calculate Test statistic and P-Value : After the experiment, you analyze the data. The “test statistic” is a number that helps you understand the difference between the two groups in terms of standard units.

For instance, let’s say:

  • The average healing time in the Drug Group is 2 hours.
  • The average healing time in the Placebo Group is 3 hours.

The test statistic helps you understand how significant this 1-hour difference is. If the groups are large and the spread of healing times in each group is small, then this difference might be significant. But if there’s a huge variation in healing times, the 1-hour difference might not be so special.

Imagine the P-value as answering this question: “If the new drug had NO real effect, what’s the probability that I’d see a difference as extreme (or more extreme) as the one I found, just by random chance?”

For instance:

  • P-value of 0.01 means there’s a 1% chance that the observed difference (or a more extreme difference) would occur if the drug had no effect. That’s pretty rare, so we might consider the drug effective.
  • P-value of 0.5 means there’s a 50% chance you’d see this difference just by chance. That’s pretty high, so we might not be convinced the drug is doing much.
  • If the P-value is less than ($α$) 0.05: the results are “statistically significant,” and they might reject the null hypothesis , believing the new drug has an effect.
  • If the P-value is greater than ($α$) 0.05: the results are not statistically significant, and they don’t reject the null hypothesis , remaining unsure if the drug has a genuine effect.

4. Example in python

For simplicity, let’s say we’re using a t-test (common for comparing means). Let’s dive into Python:

Making a Decision : “The results are statistically significant! p-value < 0.05 , The drug seems to have an effect!” If not, we’d say, “Looks like the drug isn’t as miraculous as we thought.”

5. Conclusion

Hypothesis testing is an indispensable tool in data science, allowing us to make data-driven decisions with confidence. By understanding its principles, conducting tests properly, and considering real-world applications, you can harness the power of hypothesis testing to unlock valuable insights from your data.

More Articles

F statistic formula – explained, correlation – connecting the dots, the role of correlation in data analysis, sampling and sampling distributions – a comprehensive guide on sampling and sampling distributions, law of large numbers – a deep dive into the world of statistics, central limit theorem – a deep dive into central limit theorem and its significance in statistics, similar articles, complete introduction to linear regression in r, how to implement common statistical significance tests and find the p value, logistic regression – a complete tutorial with examples in r.

Subscribe to Machine Learning Plus for high value data science content

© Machinelearningplus. All rights reserved.

use of hypothesis in machine learning

Machine Learning A-Z™: Hands-On Python & R In Data Science

Free sample videos:.

use of hypothesis in machine learning

eml header

Best Guesses: Understanding The Hypothesis in Machine Learning

Stewart Kaplan

  • February 22, 2024
  • General , Supervised Learning , Unsupervised Learning

Machine learning is a vast and complex field that has inherited many terms from other places all over the mathematical domain.

It can sometimes be challenging to get your head around all the different terminologies, never mind trying to understand how everything comes together.

In this blog post, we will focus on one particular concept: the hypothesis.

While you may think this is simple, there is a little caveat regarding machine learning.

The statistics side and the learning side.

Don’t worry; we’ll do a full breakdown below.

You’ll learn the following:

What Is a Hypothesis in Machine Learning?

  • Is This any different than the hypothesis in statistics?
  • What is the difference between the alternative hypothesis and the null?
  • Why do we restrict hypothesis space in artificial intelligence?
  • Example code performing hypothesis testing in machine learning

learning together

In machine learning, the term ‘hypothesis’ can refer to two things.

First, it can refer to the hypothesis space, the set of all possible training examples that could be used to predict or answer a new instance.

Second, it can refer to the traditional null and alternative hypotheses from statistics.

Since machine learning works so closely with statistics, 90% of the time, when someone is referencing the hypothesis, they’re referencing hypothesis tests from statistics.

Is This Any Different Than The Hypothesis In Statistics?

In statistics, the hypothesis is an assumption made about a population parameter.

The statistician’s goal is to prove it true or disprove it.

prove them wrong

This will take the form of two different hypotheses, one called the null, and one called the alternative.

Usually, you’ll establish your null hypothesis as an assumption that it equals some value.

For example, in Welch’s T-Test Of Unequal Variance, our null hypothesis is that the two means we are testing (population parameter) are equal.

This means our null hypothesis is that the two population means are the same.

We run our statistical tests, and if our p-value is significant (very low), we reject the null hypothesis.

This would mean that their population means are unequal for the two samples you are testing.

Usually, statisticians will use the significance level of .05 (a 5% risk of being wrong) when deciding what to use as the p-value cut-off.

What Is The Difference Between The Alternative Hypothesis And The Null?

The null hypothesis is our default assumption, which we are trying to prove correct.

The alternate hypothesis is usually the opposite of our null and is much broader in scope.

For most statistical tests, the null and alternative hypotheses are already defined.

You are then just trying to find “significant” evidence we can use to reject our null hypothesis.

can you prove it

These two hypotheses are easy to spot by their specific notation. The null hypothesis is usually denoted by H₀, while H₁ denotes the alternative hypothesis.

Example Code Performing Hypothesis Testing In Machine Learning

Since there are many different hypothesis tests in machine learning and data science, we will focus on one of my favorites.

This test is Welch’s T-Test Of Unequal Variance, where we are trying to determine if the population means of these two samples are different.

There are a couple of assumptions for this test, but we will ignore those for now and show the code.

You can read more about this here in our other post, Welch’s T-Test of Unequal Variance .

We see that our p-value is very low, and we reject the null hypothesis.

welch t test result with p-value

What Is The Difference Between The Biased And Unbiased Hypothesis Spaces?

The difference between the Biased and Unbiased hypothesis space is the number of possible training examples your algorithm has to predict.

The unbiased space has all of them, and the biased space only has the training examples you’ve supplied.

Since neither of these is optimal (one is too small, one is much too big), your algorithm creates generalized rules (inductive learning) to be able to handle examples it hasn’t seen before.

Here’s an example of each:

Example of The Biased Hypothesis Space In Machine Learning

The Biased Hypothesis space in machine learning is a biased subspace where your algorithm does not consider all training examples to make predictions.

This is easiest to see with an example.

Let’s say you have the following data:

Happy  and  Sunny  and  Stomach Full  = True

Whenever your algorithm sees those three together in the biased hypothesis space, it’ll automatically default to true.

This means when your algorithm sees:

Sad  and  Sunny  And  Stomach Full  = False

It’ll automatically default to False since it didn’t appear in our subspace.

This is a greedy approach, but it has some practical applications.

greedy

Example of the Unbiased Hypothesis Space In Machine Learning

The unbiased hypothesis space is a space where all combinations are stored.

We can use re-use our example above:

This would start to breakdown as

Happy  = True

Happy  and  Sunny  = True

Happy  and  Stomach Full  = True

Let’s say you have four options for each of the three choices.

This would mean our subspace would need 2^12 instances (4096) just for our little three-word problem.

This is practically impossible; the space would become huge.

subspace

So while it would be highly accurate, this has no scalability.

More reading on this idea can be found in our post, Inductive Bias In Machine Learning .

Why Do We Restrict Hypothesis Space In Artificial Intelligence?

We have to restrict the hypothesis space in machine learning. Without any restrictions, our domain becomes much too large, and we lose any form of scalability.

This is why our algorithm creates rules to handle examples that are seen in production. 

This gives our algorithms a generalized approach that will be able to handle all new examples that are in the same format.

Other Quick Machine Learning Tutorials

At EML, we have a ton of cool data science tutorials that break things down so anyone can understand them.

Below we’ve listed a few that are similar to this guide:

  • Instance-Based Learning in Machine Learning
  • Types of Data For Machine Learning
  • Verbose in Machine Learning
  • Generalization In Machine Learning
  • Epoch In Machine Learning
  • Inductive Bias in Machine Learning
  • Understanding The Hypothesis In Machine Learning
  • Zip Codes In Machine Learning
  • get_dummies() in Machine Learning
  • Bootstrapping In Machine Learning
  • X and Y in Machine Learning
  • F1 Score in Machine Learning
  • Recent Posts

Stewart Kaplan

  • Maximize Performance: How to Use Multi-Core Processors [Must-Read Tips] - September 13, 2024
  • How much do TD Ameritrade software interns make? [Discover the Real Numbers] - September 12, 2024
  • How to Use Acronis True Image Cloning Software [Top Tips] - September 12, 2024

Hypothesis in Machine Learning: Comprehensive Overview(2021)

img

Introduction

Supervised machine learning (ML) is regularly portrayed as the issue of approximating an objective capacity that maps inputs to outputs. This portrayal is described as looking through and assessing competitor hypothesis from hypothesis spaces. 

The conversation of hypothesis in machine learning can be confused for a novice, particularly when “hypothesis” has a discrete, but correlated significance in statistics and all the more comprehensively in science.

Hypothesis Space (H)

The hypothesis space utilized by an ML system is the arrangement of all hypotheses that may be returned by it. It is ordinarily characterized by a Hypothesis Language, conceivably related to a Language Bias. 

Many ML algorithms depend on some sort of search methodology: given a set of perceptions and a space of all potential hypotheses that may be thought in the hypothesis space. They see in this space for those hypotheses that adequately furnish the data or are ideal concerning some other quality standard.

ML can be portrayed as the need to utilize accessible data objects to discover a function that most reliable maps inputs to output, alluded to as function estimate, where we surmised an anonymous objective function that can most reliably map inputs to outputs on all expected perceptions from the difficult domain. An illustration of a model that approximates the performs mappings and target function of inputs to outputs is known as hypothesis testing in machine learning.

The hypothesis in machine learning of all potential hypothesis that you are looking over, paying little mind to their structure. For the wellbeing of accommodation, the hypothesis class is normally compelled to be just each sort of function or model in turn, since learning techniques regularly just work on each type at a time. This doesn’t need to be the situation, however:

  • Hypothesis classes don’t need to comprise just one kind of function. If you’re looking through exponential, quadratic, and overall linear functions, those are what your joined hypothesis class contains.
  • Hypothesis classes additionally don’t need to comprise of just straightforward functions. If you figure out how to look over all piecewise-tanh2 functions, those functions are what your hypothesis class incorporates.

The enormous trade-off is that the bigger your hypothesis class in   machine learning, the better the best hypothesis models the basic genuine function, yet the harder it is to locate that best hypothesis. This is identified with the bias-variance trade-off.

  • Hypothesis (h)

A hypothesis function in machine learning is best describes the target. The hypothesis that an algorithm would concoct relies on the data and relies on the bias and restrictions that we have forced on the data.

The hypothesis formula in machine learning:

  • y  is range
  • m  changes in y divided by change in x
  • x  is domain
  • b  is intercept

The purpose of restricting hypothesis space in machine learning is so that these can fit well with the general data that is needed by the user. It checks the reality or deception of observations or inputs and examinations them appropriately. Subsequently, it is extremely helpful and it plays out the valuable function of mapping all the inputs till they come out as outputs. Consequently, the target functions are deliberately examined and restricted dependent on the outcomes (regardless of whether they are free of bias), in ML.

The hypothesis in machine learning space and inductive bias in machine learning is that the hypothesis space is a collection of valid Hypothesis, for example, every single desirable function, on the opposite side the inductive bias (otherwise called learning bias) of a learning algorithm is the series of expectations that the learner uses to foresee outputs of given sources of inputs that it has not experienced. Regression and Classification are a kind of realizing which relies upon continuous-valued and discrete-valued sequentially. This sort of issues (learnings) is called inductive learning issues since we distinguish a function by inducting it on data.

In the Maximum a Posteriori or MAP hypothesis in machine learning, enhancement gives a Bayesian probability structure to fitting model parameters to training data and another option and sibling may be a more normal Maximum Likelihood Estimation system. MAP learning chooses a solitary in all probability theory given the data. The hypothesis in machine learning earlier is as yet utilized and the technique is regularly more manageable than full Bayesian learning. 

Bayesian techniques can be utilized to decide the most plausible hypothesis in machine learning given the data the MAP hypothesis. This is the ideal hypothesis as no other hypothesis is more probable.

Hypothesis in machine learning or ML the applicant model that approximates a target function for mapping instances of inputs to outputs.

Hypothesis in statistics probabilistic clarification about the presence of a connection between observations. 

Hypothesis in science is a temporary clarification that fits the proof and can be disproved or confirmed. We can see that a hypothesis in machine learning draws upon the meaning of the hypothesis all the more extensively in science.

There are no right or wrong ways of learning AI and ML technologies – the more, the better! These valuable resources can be the starting point for your journey on how to learn Artificial Intelligence and Machine Learning. Do pursuing AI and ML interest you? If you want to step into the world of emerging tech, you can accelerate your career with this  Machine Learning And AI Courses   by Jigsaw Academy.

  • XGBoost Algorithm: An Easy Overview For 2021

tag-img

Fill in the details to know more

facebook

PEOPLE ALSO READ

use of hypothesis in machine learning

Related Articles

use of hypothesis in machine learning

From The Eyes Of Emerging Technologies: IPL Through The Ages

April 29, 2023

 width=

Personalized Teaching with AI: Revolutionizing Traditional Teaching Methods

April 28, 2023

img

Metaverse: The Virtual Universe and its impact on the World of Finance

April 13, 2023

img

Artificial Intelligence – Learning To Manage The Mind Created By The Human Mind!

March 22, 2023

use of hypothesis in machine learning

Wake Up to the Importance of Sleep: Celebrating World Sleep Day!

March 18, 2023

use of hypothesis in machine learning

Operations Management and AI: How Do They Work?

March 15, 2023

img

How Does BYOP(Bring Your Own Project) Help In Building Your Portfolio?

use of hypothesis in machine learning

What Are the Ethics in Artificial Intelligence (AI)?

November 25, 2022

epoch in machine learning

What is Epoch in Machine Learning?| UNext

November 24, 2022

use of hypothesis in machine learning

The Impact Of Artificial Intelligence (AI) in Cloud Computing

November 18, 2022

use of hypothesis in machine learning

Role of Artificial Intelligence and Machine Learning in Supply Chain Management 

November 11, 2022

use of hypothesis in machine learning

Best Python Libraries for Machine Learning in 2022

November 7, 2022

share

Are you ready to build your own career?

arrow

Query? Ask Us

use of hypothesis in machine learning

Get In Touch

Experiential upskilling programs tailored to your organization's needs.

Add your details:

By proceeding, you agree to our privacy policy and also agree to receive information from UNext through WhatsApp & other means of communication.

& other means of communication. --> By proceeding, you agree to our privacy policy and also agree to receive information from UNext through WhatsApp & other means of communication.

Subscribe To Our Newsletter

Upgrade your inbox with our curated newletters once every month. We appreciate your support and will make sure to keep your subscription worthwhile

Enter Your Details ×

Machine Learning Tutorial

  • Machine Learning Basics
  • Machine Learning - Home
  • Machine Learning - Introduction
  • Machine Learning - Getting Started
  • Machine Learning - Basic Concepts
  • Machine Learning - Ecosystem
  • Machine Learning - Python Libraries
  • Machine Learning - Applications
  • Machine Learning - Life Cycle
  • Machine Learning - Required Skills
  • Machine Learning - Implementation
  • Machine Learning - Challenges & Common Issues
  • Machine Learning - Limitations
  • Machine Learning - Reallife Examples
  • Machine Learning - Data Structure
  • Machine Learning - Mathematics
  • Machine Learning - Artificial Intelligence
  • Machine Learning - Neural Networks
  • Machine Learning - Deep Learning
  • Machine Learning - Getting Datasets
  • Machine Learning - Categorical Data
  • Machine Learning - Data Loading
  • Machine Learning - Data Understanding
  • Machine Learning - Data Preparation
  • Machine Learning - Models
  • Machine Learning - Supervised
  • Machine Learning - Unsupervised
  • Machine Learning - Semi-supervised
  • Machine Learning - Reinforcement
  • Machine Learning - Supervised vs. Unsupervised
  • What Today’s AI Can Do?
  • What is Machine Learning?
  • Machine Learning - Categories
  • Machine Learning - Scikit-learn Algorithm
  • Machine Learning - Conclusion
  • Machine Learning Data Visualization
  • Machine Learning - Data Visualization
  • Machine Learning - Histograms
  • Machine Learning - Density Plots
  • Machine Learning - Box and Whisker Plots
  • Machine Learning - Correlation Matrix Plots
  • Machine Learning - Scatter Matrix Plots
  • Statistics for Machine Learning
  • Machine Learning - Statistics
  • Machine Learning - Mean, Median, Mode
  • Machine Learning - Standard Deviation
  • Machine Learning - Percentiles
  • Machine Learning - Data Distribution
  • Machine Learning - Skewness and Kurtosis
  • Machine Learning - Bias and Variance

Machine Learning - Hypothesis

  • Regression Analysis In ML
  • Machine Learning - Regression Analysis
  • Machine Learning - Linear Regression
  • Machine Learning - Simple Linear Regression
  • Machine Learning - Multiple Linear Regression
  • Machine Learning - Polynomial Regression
  • Classification Algorithms In ML
  • Machine Learning - Classification Algorithms
  • Machine Learning - Logistic Regression
  • Machine Learning - K-Nearest Neighbors (KNN)
  • Machine Learning - Naïve Bayes Algorithm
  • Machine Learning - Decision Tree Algorithm
  • Machine Learning - Support Vector Machine
  • Machine Learning - Random Forest
  • Machine Learning - Confusion Matrix
  • Machine Learning - Stochastic Gradient Descent
  • Clustering Algorithms In ML
  • Machine Learning - Clustering Algorithms
  • Machine Learning - Centroid-Based Clustering
  • Machine Learning - K-Means Clustering
  • Machine Learning - K-Medoids Clustering
  • Machine Learning - Mean-Shift Clustering
  • Machine Learning - Hierarchical Clustering
  • Machine Learning - Density-Based Clustering
  • Machine Learning - DBSCAN Clustering
  • Machine Learning - OPTICS Clustering
  • Machine Learning - HDBSCAN Clustering
  • Machine Learning - BIRCH Clustering
  • Machine Learning - Affinity Propagation
  • Machine Learning - Distribution-Based Clustering
  • Machine Learning - Agglomerative Clustering
  • Dimensionality Reduction In ML
  • Machine Learning - Dimensionality Reduction
  • Machine Learning - Feature Selection
  • Machine Learning - Feature Extraction
  • Machine Learning - Backward Elimination
  • Machine Learning - Forward Feature Construction
  • Machine Learning - High Correlation Filter
  • Machine Learning - Low Variance Filter
  • Machine Learning - Missing Values Ratio
  • Machine Learning - Principal Component Analysis
  • Machine Learning Miscellaneous
  • Machine Learning - Performance Metrics
  • Machine Learning - Automatic Workflows
  • Machine Learning - Boost Model Performance
  • Machine Learning - Gradient Boosting
  • Machine Learning - Bootstrap Aggregation (Bagging)
  • Machine Learning - Cross Validation
  • Machine Learning - AUC-ROC Curve
  • Machine Learning - Grid Search
  • Machine Learning - Data Scaling
  • Machine Learning - Train and Test
  • Machine Learning - Association Rules
  • Machine Learning - Apriori Algorithm
  • Machine Learning - Gaussian Discriminant Analysis
  • Machine Learning - Cost Function
  • Machine Learning - Bayes Theorem
  • Machine Learning - Precision and Recall
  • Machine Learning - Adversarial
  • Machine Learning - Stacking
  • Machine Learning - Epoch
  • Machine Learning - Perceptron
  • Machine Learning - Regularization
  • Machine Learning - Overfitting
  • Machine Learning - P-value
  • Machine Learning - Entropy
  • Machine Learning - MLOps
  • Machine Learning - Data Leakage
  • Machine Learning - Resources
  • Machine Learning - Quick Guide
  • Machine Learning - Useful Resources
  • Machine Learning - Discussion
  • Selected Reading
  • UPSC IAS Exams Notes
  • Developer's Best Practices
  • Questions and Answers
  • Effective Resume Writing
  • HR Interview Questions
  • Computer Glossary

In machine learning, a hypothesis is a proposed explanation or solution for a problem. It is a tentative assumption or idea that can be tested and validated using data. In supervised learning, the hypothesis is the model that the algorithm is trained on to make predictions on unseen data.

The hypothesis is generally expressed as a function that maps input data to output labels. In other words, it defines the relationship between the input and output variables. The goal of machine learning is to find the best possible hypothesis that can generalize well to unseen data.

The process of finding the best hypothesis is called model training or learning. During the training process, the algorithm adjusts the model parameters to minimize the error or loss function, which measures the difference between the predicted output and the actual output.

Once the model is trained, it can be used to make predictions on new data. However, it is important to evaluate the performance of the model before using it in the real world. This is done by testing the model on a separate validation set or using cross-validation techniques.

Properties of a Good Hypothesis

The hypothesis plays a critical role in the success of a machine learning model. A good hypothesis should have the following properties −

Generalization − The model should be able to make accurate predictions on unseen data.

Simplicity − The model should be simple and interpretable, so that it is easier to understand and explain.

Robustness − The model should be able to handle noise and outliers in the data.

Scalability − The model should be able to handle large amounts of data efficiently.

There are many types of machine learning algorithms that can be used to generate hypotheses, including linear regression, logistic regression, decision trees, support vector machines, neural networks, and more.

Hold on, while it loads...

name

Evaluating Hypotheses in Machine Learning: A Comprehensive Guide

Learn how to evaluate hypotheses in machine learning, including types of hypotheses, evaluation metrics, and common pitfalls to avoid. Improve your ML model's performance with this in-depth guide.

Create an image featuring JavaScript code snippets and interview-related icons or graphics. Use a color scheme of yellows and blues. Include the title '7 Essential JavaScript Interview Questions for Freshers'.

Create an image featuring JavaScript code snippets and interview-related icons or graphics. Use a color scheme of yellows and blues. Include the title '7 Essential JavaScript Interview Questions for Freshers'.

Introduction

Machine learning is a crucial aspect of artificial intelligence that enables machines to learn from data and make predictions or decisions. The process of machine learning involves training a model on a dataset, and then using that model to make predictions on new, unseen data. However, before deploying a machine learning model, it is essential to evaluate its performance to ensure that it is accurate and reliable. One crucial step in this evaluation process is hypothesis testing.

In this blog post, we will delve into the world of hypothesis testing in machine learning, exploring what hypotheses are, why they are essential, and how to evaluate them. We will also discuss the different types of hypotheses, common pitfalls to avoid, and best practices for hypothesis testing.

What are Hypotheses in Machine Learning?

In machine learning, a hypothesis is a statement that proposes a possible explanation for a phenomenon or a problem. It is a conjecture that is made about a population parameter, and it is used as a basis for further investigation. In the context of machine learning, hypotheses are used to define the problem that we are trying to solve.

For example, let's say we are building a machine learning model to predict the prices of houses based on their features, such as the number of bedrooms, square footage, and location. A possible hypothesis could be: "The price of a house is directly proportional to its square footage." This hypothesis proposes a possible relationship between the price of a house and its square footage.

Why are Hypotheses Essential in Machine Learning?

Hypotheses are essential in machine learning because they provide a framework for understanding the problem that we are trying to solve. They help us to identify the key variables that are relevant to the problem, and they provide a basis for evaluating the performance of our machine learning model.

Without a clear hypothesis, it is difficult to develop an effective machine learning model. A hypothesis helps us to:

  • Identify the key variables that are relevant to the problem
  • Develop a clear understanding of the problem that we are trying to solve
  • Evaluate the performance of our machine learning model
  • Refine our model and improve its accuracy

Types of Hypotheses in Machine Learning

There are two main types of hypotheses in machine learning: null hypotheses and alternative hypotheses.

Null Hypothesis

A null hypothesis is a hypothesis that proposes that there is no significant difference or relationship between variables. It is a hypothesis of no effect or no difference. For example, let's say we are building a machine learning model to predict the prices of houses based on their features. A null hypothesis could be: "There is no significant relationship between the price of a house and its square footage."

Alternative Hypothesis

An alternative hypothesis is a hypothesis that proposes that there is a significant difference or relationship between variables. It is a hypothesis of an effect or a difference. For example, let's say we are building a machine learning model to predict the prices of houses based on their features. An alternative hypothesis could be: "There is a significant positive relationship between the price of a house and its square footage."

Evaluating Hypotheses in Machine Learning

Evaluating hypotheses in machine learning involves testing the null hypothesis against the alternative hypothesis. This is typically done using statistical methods, such as t-tests, ANOVA, and regression analysis.

Here are the general steps involved in evaluating hypotheses in machine learning:

  • Formulate the null and alternative hypotheses : Clearly define the null and alternative hypotheses that you want to test.
  • Collect and prepare the data : Collect the data that you will use to test the hypotheses. Ensure that the data is clean, relevant, and representative of the population.
  • Choose a statistical method : Select a suitable statistical method to test the hypotheses. This could be a t-test, ANOVA, regression analysis, or another method.
  • Test the hypotheses : Use the chosen statistical method to test the null hypothesis against the alternative hypothesis.
  • Interpret the results : Interpret the results of the hypothesis test. If the null hypothesis is rejected, it suggests that there is a significant relationship between the variables. If the null hypothesis is not rejected, it suggests that there is no significant relationship between the variables.

Common Pitfalls to Avoid in Hypothesis Testing

Here are some common pitfalls to avoid in hypothesis testing:

  • Overfitting : Overfitting occurs when a model is too complex and performs well on the training data but poorly on new, unseen data. To avoid overfitting, use techniques such as regularization, early stopping, and cross-validation.
  • Underfitting : Underfitting occurs when a model is too simple and fails to capture the underlying patterns in the data. To avoid underfitting, use techniques such as feature engineering, hyperparameter tuning, and model selection.
  • Data leakage : Data leakage occurs when the model is trained on data that it will also be tested on. To avoid data leakage, use techniques such as cross-validation and walk-forward optimization.
  • P-hacking : P-hacking occurs when a researcher selectively reports the results of multiple hypothesis tests to find a significant result. To avoid p-hacking, use techniques such as preregistration and replication.

Best Practices for Hypothesis Testing in Machine Learning

Here are some best practices for hypothesis testing in machine learning:

  • Clearly define the hypotheses : Clearly define the null and alternative hypotheses that you want to test.
  • Use a suitable statistical method : Choose a suitable statistical method to test the hypotheses.
  • Use cross-validation : Use cross-validation to evaluate the performance of the model on unseen data.
  • Avoid overfitting and underfitting : Use techniques such as regularization, early stopping, and feature engineering to avoid overfitting and underfitting.
  • Document the results : Document the results of the hypothesis test, including the statistical method used, the results, and any conclusions drawn.

Evaluating hypotheses is a crucial step in machine learning that helps us to understand the problem that we are trying to solve and to evaluate the performance of our machine learning model. By following the best practices outlined in this blog post, you can ensure that your hypothesis testing is rigorous, reliable, and effective.

Remember to clearly define the null and alternative hypotheses, choose a suitable statistical method, and avoid common pitfalls such as overfitting, underfitting, data leakage, and p-hacking. By doing so, you can develop machine learning models that are accurate, reliable, and effective.

  • [1] James, G., Witten, D., Hastie, T., & Tibshirani, R. (2013). An Introduction to Statistical Learning: With Applications in R. Springer.
  • [2] Bishop, C. M. (2006). Pattern Recognition and Machine Learning. Springer.
  • [3] Han, J., Pei, J., & Kamber, M. (2012). Data Mining: Concepts and Techniques. Morgan Kaufmann.

I hope this helps! Let me know if you need any further assistance.

Interview scenario with a laptop and a man displaying behavioral interview questions

What’s a Hypothesis Space?

Last updated: March 18, 2024

use of hypothesis in machine learning

  • Math and Logic

announcement - icon

Baeldung Pro comes with both absolutely No-Ads as well as finally with Dark Mode , for a clean learning experience:

>> Explore a clean Baeldung

Once the early-adopter seats are all used, the price will go up and stay at $33/year.

1. Introduction

Machine-learning algorithms come with implicit or explicit assumptions about the actual patterns in the data. Mathematically, this means that each algorithm can learn a specific family of models, and that family goes by the name of the hypothesis space.

In this tutorial, we’ll talk about hypothesis spaces and how to choose the right one for the data at hand.

2. Hypothesis Spaces

Let’s say that we have a binary classification task and that the data are two-dimensional. Our goal is to find a model that classifies objects as positive or negative. Applying Logistic Regression , we can get the models of the form:

which estimate the probability that the object at hand is positive.

2.1. Hypotheses and Assumptions

The underlying assumption of hypotheses ( 1 ) is that the boundary separating the positive from negative objects is a straight line. So, every hypothesis from this space corresponds to a straight line in a 2D plane. For instance:

Two Classification Hypotheses

2.2. Regression

3. expressivity of a hypothesis space.

We could informally say that one hypothesis space is more expressive than another if its hypotheses are more diverse and complex.

We may underfit the data if our algorithm’s hypothesis space isn’t expressive enough. For instance, linear hypotheses aren’t particularly good options if the actual data are extremely non-linear:

Non-linear Data

So, training an algorithm that has a very expressive space increases the chance of completely capturing the patterns in the data. However, it also increases the risk of overfitting. For instance, a space containing the hypotheses of the form:

would start modelling the noise, which we see from its decision boundary:

A too complex hypothesis

Such models would generalize poorly to unseen data.

3.1. Expressivity vs. Interpretability

Additionally, even if a complex hypothesis has a good generalization capability, it may be unusable in practice because it’s too complicated to understand or compute. What’s more, intricated hypotheses offer limited insight into the real-world process that generated the data. For example, a quadratic model:

4. How to Choose the Hypothesis Space?

We need to find the right balance between expressivity and simplicity. Unfortunately, that’s easier said than done. Most of the time, we need to rely on our intuition about the data.

So, we should start by exploring the dataset, using visualizations as much as possible. For instance, we can conclude that a straight line isn’t likely to be an adequate boundary for the above classification data. However, a high-order curve would probably be too complex even though it might split the dataset into two classes without an error.

A second-degree curve might be the compromise we seek, but we aren’t sure. So, we start with the space of quadratic hypotheses:

We get a model whose decision boundary appears to be a good fit even though it misclassifies some objects:

An adequate hypothesis

Since we’re satisfied with the model, we can stop here. If that hadn’t been the case, we could have tried a space of cubic models. The idea would be to iteratively try incrementally complex families until finding a model that both performs well and is easy to understand.

4. Conclusion

In this article, we talked about hypotheses spaces in machine learning. An algorithm’s hypothesis space contains all the models it can learn from any dataset.

The algorithms with too expressive spaces can generalize poorly to unseen data and be too complex to understand, whereas those with overly simple hypotheses may underfit the data. So, when applying machine-learning algorithms in practice, we need to find the right balance between expressivity and simplicity.

  • Data Science
  • Data Analysis
  • Data Visualization
  • Machine Learning
  • Deep Learning
  • Computer Vision
  • Artificial Intelligence
  • AI ML DS Interview Series
  • AI ML DS Projects series
  • Data Engineering
  • Web Scrapping

Understanding Hypothesis Testing

Hypothesis testing involves formulating assumptions about population parameters based on sample statistics and rigorously evaluating these assumptions against empirical evidence. This article sheds light on the significance of hypothesis testing and the critical steps involved in the process.

What is Hypothesis Testing?

A hypothesis is an assumption or idea, specifically a statistical claim about an unknown population parameter. For example, a judge assumes a person is innocent and verifies this by reviewing evidence and hearing testimony before reaching a verdict.

Hypothesis testing is a statistical method that is used to make a statistical decision using experimental data. Hypothesis testing is basically an assumption that we make about a population parameter. It evaluates two mutually exclusive statements about a population to determine which statement is best supported by the sample data. 

To test the validity of the claim or assumption about the population parameter:

  • A sample is drawn from the population and analyzed.
  • The results of the analysis are used to decide whether the claim is true or not.
Example: You say an average height in the class is 30 or a boy is taller than a girl. All of these is an assumption that we are assuming, and we need some statistical way to prove these. We need some mathematical conclusion whatever we are assuming is true.

Defining Hypotheses

  • Null hypothesis (H 0 ): In statistics, the null hypothesis is a general statement or default position that there is no relationship between two measured cases or no relationship among groups. In other words, it is a basic assumption or made based on the problem knowledge. Example : A company’s mean production is 50 units/per da H 0 : [Tex]\mu [/Tex] = 50.
  • Alternative hypothesis (H 1 ): The alternative hypothesis is the hypothesis used in hypothesis testing that is contrary to the null hypothesis.  Example: A company’s production is not equal to 50 units/per day i.e. H 1 : [Tex]\mu [/Tex] [Tex]\ne [/Tex] 50.

Key Terms of Hypothesis Testing

  • Level of significance : It refers to the degree of significance in which we accept or reject the null hypothesis. 100% accuracy is not possible for accepting a hypothesis, so we, therefore, select a level of significance that is usually 5%. This is normally denoted with  [Tex]\alpha[/Tex] and generally, it is 0.05 or 5%, which means your output should be 95% confident to give a similar kind of result in each sample.
  • P-value: The P value , or calculated probability, is the probability of finding the observed/extreme results when the null hypothesis(H0) of a study-given problem is true. If your P-value is less than the chosen significance level then you reject the null hypothesis i.e. accept that your sample claims to support the alternative hypothesis.
  • Test Statistic: The test statistic is a numerical value calculated from sample data during a hypothesis test, used to determine whether to reject the null hypothesis. It is compared to a critical value or p-value to make decisions about the statistical significance of the observed results.
  • Critical value : The critical value in statistics is a threshold or cutoff point used to determine whether to reject the null hypothesis in a hypothesis test.
  • Degrees of freedom: Degrees of freedom are associated with the variability or freedom one has in estimating a parameter. The degrees of freedom are related to the sample size and determine the shape.

Why do we use Hypothesis Testing?

Hypothesis testing is an important procedure in statistics. Hypothesis testing evaluates two mutually exclusive population statements to determine which statement is most supported by sample data. When we say that the findings are statistically significant, thanks to hypothesis testing. 

One-Tailed and Two-Tailed Test

One tailed test focuses on one direction, either greater than or less than a specified value. We use a one-tailed test when there is a clear directional expectation based on prior knowledge or theory. The critical region is located on only one side of the distribution curve. If the sample falls into this critical region, the null hypothesis is rejected in favor of the alternative hypothesis.

One-Tailed Test

There are two types of one-tailed test:

  • Left-Tailed (Left-Sided) Test: The alternative hypothesis asserts that the true parameter value is less than the null hypothesis. Example: H 0 ​: [Tex]\mu \geq 50 [/Tex] and H 1 : [Tex]\mu < 50 [/Tex]
  • Right-Tailed (Right-Sided) Test : The alternative hypothesis asserts that the true parameter value is greater than the null hypothesis. Example: H 0 : [Tex]\mu \leq50 [/Tex] and H 1 : [Tex]\mu > 50 [/Tex]

Two-Tailed Test

A two-tailed test considers both directions, greater than and less than a specified value.We use a two-tailed test when there is no specific directional expectation, and want to detect any significant difference.

Example: H 0 : [Tex]\mu = [/Tex] 50 and H 1 : [Tex]\mu \neq 50 [/Tex]

To delve deeper into differences into both types of test: Refer to link

What are Type 1 and Type 2 errors in Hypothesis Testing?

In hypothesis testing, Type I and Type II errors are two possible errors that researchers can make when drawing conclusions about a population based on a sample of data. These errors are associated with the decisions made regarding the null hypothesis and the alternative hypothesis.

  • Type I error: When we reject the null hypothesis, although that hypothesis was true. Type I error is denoted by alpha( [Tex]\alpha [/Tex] ).
  • Type II errors : When we accept the null hypothesis, but it is false. Type II errors are denoted by beta( [Tex]\beta [/Tex] ).


Null Hypothesis is True

Null Hypothesis is False

Null Hypothesis is True (Accept)

Correct Decision

Type II Error (False Negative)

Alternative Hypothesis is True (Reject)

Type I Error (False Positive)

Correct Decision

How does Hypothesis Testing work?

Step 1: define null and alternative hypothesis.

State the null hypothesis ( [Tex]H_0 [/Tex] ), representing no effect, and the alternative hypothesis ( [Tex]H_1 [/Tex] ​), suggesting an effect or difference.

We first identify the problem about which we want to make an assumption keeping in mind that our assumption should be contradictory to one another, assuming Normally distributed data.

Step 2 – Choose significance level

Select a significance level ( [Tex]\alpha [/Tex] ), typically 0.05, to determine the threshold for rejecting the null hypothesis. It provides validity to our hypothesis test, ensuring that we have sufficient data to back up our claims. Usually, we determine our significance level beforehand of the test. The p-value is the criterion used to calculate our significance value.

Step 3 – Collect and Analyze data.

Gather relevant data through observation or experimentation. Analyze the data using appropriate statistical methods to obtain a test statistic.

Step 4-Calculate Test Statistic

The data for the tests are evaluated in this step we look for various scores based on the characteristics of data. The choice of the test statistic depends on the type of hypothesis test being conducted.

There are various hypothesis tests, each appropriate for various goal to calculate our test. This could be a Z-test , Chi-square , T-test , and so on.

  • Z-test : If population means and standard deviations are known. Z-statistic is commonly used.
  • t-test : If population standard deviations are unknown. and sample size is small than t-test statistic is more appropriate.
  • Chi-square test : Chi-square test is used for categorical data or for testing independence in contingency tables
  • F-test : F-test is often used in analysis of variance (ANOVA) to compare variances or test the equality of means across multiple groups.

We have a smaller dataset, So, T-test is more appropriate to test our hypothesis.

T-statistic is a measure of the difference between the means of two groups relative to the variability within each group. It is calculated as the difference between the sample means divided by the standard error of the difference. It is also known as the t-value or t-score.

Step 5 – Comparing Test Statistic:

In this stage, we decide where we should accept the null hypothesis or reject the null hypothesis. There are two ways to decide where we should accept or reject the null hypothesis.

Method A: Using Crtical values

Comparing the test statistic and tabulated critical value we have,

  • If Test Statistic>Critical Value: Reject the null hypothesis.
  • If Test Statistic≤Critical Value: Fail to reject the null hypothesis.

Note: Critical values are predetermined threshold values that are used to make a decision in hypothesis testing. To determine critical values for hypothesis testing, we typically refer to a statistical distribution table , such as the normal distribution or t-distribution tables based on.

Method B: Using P-values

We can also come to an conclusion using the p-value,

  • If the p-value is less than or equal to the significance level i.e. ( [Tex]p\leq\alpha [/Tex] ), you reject the null hypothesis. This indicates that the observed results are unlikely to have occurred by chance alone, providing evidence in favor of the alternative hypothesis.
  • If the p-value is greater than the significance level i.e. ( [Tex]p\geq \alpha[/Tex] ), you fail to reject the null hypothesis. This suggests that the observed results are consistent with what would be expected under the null hypothesis.

Note : The p-value is the probability of obtaining a test statistic as extreme as, or more extreme than, the one observed in the sample, assuming the null hypothesis is true. To determine p-value for hypothesis testing, we typically refer to a statistical distribution table , such as the normal distribution or t-distribution tables based on.

Step 7- Interpret the Results

At last, we can conclude our experiment using method A or B.

Calculating test statistic

To validate our hypothesis about a population parameter we use statistical functions . We use the z-score, p-value, and level of significance(alpha) to make evidence for our hypothesis for normally distributed data .

1. Z-statistics:

When population means and standard deviations are known.

[Tex]z = \frac{\bar{x} – \mu}{\frac{\sigma}{\sqrt{n}}}[/Tex]

  • [Tex]\bar{x} [/Tex] is the sample mean,
  • μ represents the population mean, 
  • σ is the standard deviation
  • and n is the size of the sample.

2. T-Statistics

T test is used when n<30,

t-statistic calculation is given by:

[Tex]t=\frac{x̄-μ}{s/\sqrt{n}} [/Tex]

  • t = t-score,
  • x̄ = sample mean
  • μ = population mean,
  • s = standard deviation of the sample,
  • n = sample size

3. Chi-Square Test

Chi-Square Test for Independence categorical Data (Non-normally distributed) using:

[Tex]\chi^2 = \sum \frac{(O_{ij} – E_{ij})^2}{E_{ij}}[/Tex]

  • [Tex]O_{ij}[/Tex] is the observed frequency in cell [Tex]{ij} [/Tex]
  • i,j are the rows and columns index respectively.
  • [Tex]E_{ij}[/Tex] is the expected frequency in cell [Tex]{ij}[/Tex] , calculated as : [Tex]\frac{{\text{{Row total}} \times \text{{Column total}}}}{{\text{{Total observations}}}}[/Tex]

Real life Examples of Hypothesis Testing

Let’s examine hypothesis testing using two real life situations,

Case A: D oes a New Drug Affect Blood Pressure?

Imagine a pharmaceutical company has developed a new drug that they believe can effectively lower blood pressure in patients with hypertension. Before bringing the drug to market, they need to conduct a study to assess its impact on blood pressure.

  • Before Treatment: 120, 122, 118, 130, 125, 128, 115, 121, 123, 119
  • After Treatment: 115, 120, 112, 128, 122, 125, 110, 117, 119, 114

Step 1 : Define the Hypothesis

  • Null Hypothesis : (H 0 )The new drug has no effect on blood pressure.
  • Alternate Hypothesis : (H 1 )The new drug has an effect on blood pressure.

Step 2: Define the Significance level

Let’s consider the Significance level at 0.05, indicating rejection of the null hypothesis.

If the evidence suggests less than a 5% chance of observing the results due to random variation.

Step 3 : Compute the test statistic

Using paired T-test analyze the data to obtain a test statistic and a p-value.

The test statistic (e.g., T-statistic) is calculated based on the differences between blood pressure measurements before and after treatment.

t = m/(s/√n)

  • m  = mean of the difference i.e X after, X before
  • s  = standard deviation of the difference (d) i.e d i ​= X after, i ​− X before,
  • n  = sample size,

then, m= -3.9, s= 1.8 and n= 10

we, calculate the , T-statistic = -9 based on the formula for paired t test

Step 4: Find the p-value

The calculated t-statistic is -9 and degrees of freedom df = 9, you can find the p-value using statistical software or a t-distribution table.

thus, p-value = 8.538051223166285e-06

Step 5: Result

  • If the p-value is less than or equal to 0.05, the researchers reject the null hypothesis.
  • If the p-value is greater than 0.05, they fail to reject the null hypothesis.

Conclusion: Since the p-value (8.538051223166285e-06) is less than the significance level (0.05), the researchers reject the null hypothesis. There is statistically significant evidence that the average blood pressure before and after treatment with the new drug is different.

Python Implementation of Case A

Let’s create hypothesis testing with python, where we are testing whether a new drug affects blood pressure. For this example, we will use a paired T-test. We’ll use the scipy.stats library for the T-test.

Scipy is a mathematical library in Python that is mostly used for mathematical equations and computations.

We will implement our first real life problem via python,

import numpy as np from scipy import stats # Data before_treatment = np . array ([ 120 , 122 , 118 , 130 , 125 , 128 , 115 , 121 , 123 , 119 ]) after_treatment = np . array ([ 115 , 120 , 112 , 128 , 122 , 125 , 110 , 117 , 119 , 114 ]) # Step 1: Null and Alternate Hypotheses # Null Hypothesis: The new drug has no effect on blood pressure. # Alternate Hypothesis: The new drug has an effect on blood pressure. null_hypothesis = "The new drug has no effect on blood pressure." alternate_hypothesis = "The new drug has an effect on blood pressure." # Step 2: Significance Level alpha = 0.05 # Step 3: Paired T-test t_statistic , p_value = stats . ttest_rel ( after_treatment , before_treatment ) # Step 4: Calculate T-statistic manually m = np . mean ( after_treatment - before_treatment ) s = np . std ( after_treatment - before_treatment , ddof = 1 ) # using ddof=1 for sample standard deviation n = len ( before_treatment ) t_statistic_manual = m / ( s / np . sqrt ( n )) # Step 5: Decision if p_value <= alpha : decision = "Reject" else : decision = "Fail to reject" # Conclusion if decision == "Reject" : conclusion = "There is statistically significant evidence that the average blood pressure before and after treatment with the new drug is different." else : conclusion = "There is insufficient evidence to claim a significant difference in average blood pressure before and after treatment with the new drug." # Display results print ( "T-statistic (from scipy):" , t_statistic ) print ( "P-value (from scipy):" , p_value ) print ( "T-statistic (calculated manually):" , t_statistic_manual ) print ( f "Decision: { decision } the null hypothesis at alpha= { alpha } ." ) print ( "Conclusion:" , conclusion )

T-statistic (from scipy): -9.0 P-value (from scipy): 8.538051223166285e-06 T-statistic (calculated manually): -9.0 Decision: Reject the null hypothesis at alpha=0.05. Conclusion: There is statistically significant evidence that the average blood pressure before and after treatment with the new drug is different.

In the above example, given the T-statistic of approximately -9 and an extremely small p-value, the results indicate a strong case to reject the null hypothesis at a significance level of 0.05. 

  • The results suggest that the new drug, treatment, or intervention has a significant effect on lowering blood pressure.
  • The negative T-statistic indicates that the mean blood pressure after treatment is significantly lower than the assumed population mean before treatment.

Case B : Cholesterol level in a population

Data: A sample of 25 individuals is taken, and their cholesterol levels are measured.

Cholesterol Levels (mg/dL): 205, 198, 210, 190, 215, 205, 200, 192, 198, 205, 198, 202, 208, 200, 205, 198, 205, 210, 192, 205, 198, 205, 210, 192, 205.

Populations Mean = 200

Population Standard Deviation (σ): 5 mg/dL(given for this problem)

Step 1: Define the Hypothesis

  • Null Hypothesis (H 0 ): The average cholesterol level in a population is 200 mg/dL.
  • Alternate Hypothesis (H 1 ): The average cholesterol level in a population is different from 200 mg/dL.

As the direction of deviation is not given , we assume a two-tailed test, and based on a normal distribution table, the critical values for a significance level of 0.05 (two-tailed) can be calculated through the z-table and are approximately -1.96 and 1.96.

The test statistic is calculated by using the z formula Z = [Tex](203.8 – 200) / (5 \div \sqrt{25}) [/Tex] ​ and we get accordingly , Z =2.039999999999992.

Step 4: Result

Since the absolute value of the test statistic (2.04) is greater than the critical value (1.96), we reject the null hypothesis. And conclude that, there is statistically significant evidence that the average cholesterol level in the population is different from 200 mg/dL

Python Implementation of Case B

import scipy.stats as stats import math import numpy as np # Given data sample_data = np . array ( [ 205 , 198 , 210 , 190 , 215 , 205 , 200 , 192 , 198 , 205 , 198 , 202 , 208 , 200 , 205 , 198 , 205 , 210 , 192 , 205 , 198 , 205 , 210 , 192 , 205 ]) population_std_dev = 5 population_mean = 200 sample_size = len ( sample_data ) # Step 1: Define the Hypotheses # Null Hypothesis (H0): The average cholesterol level in a population is 200 mg/dL. # Alternate Hypothesis (H1): The average cholesterol level in a population is different from 200 mg/dL. # Step 2: Define the Significance Level alpha = 0.05 # Two-tailed test # Critical values for a significance level of 0.05 (two-tailed) critical_value_left = stats . norm . ppf ( alpha / 2 ) critical_value_right = - critical_value_left # Step 3: Compute the test statistic sample_mean = sample_data . mean () z_score = ( sample_mean - population_mean ) / \ ( population_std_dev / math . sqrt ( sample_size )) # Step 4: Result # Check if the absolute value of the test statistic is greater than the critical values if abs ( z_score ) > max ( abs ( critical_value_left ), abs ( critical_value_right )): print ( "Reject the null hypothesis." ) print ( "There is statistically significant evidence that the average cholesterol level in the population is different from 200 mg/dL." ) else : print ( "Fail to reject the null hypothesis." ) print ( "There is not enough evidence to conclude that the average cholesterol level in the population is different from 200 mg/dL." )

Reject the null hypothesis. There is statistically significant evidence that the average cholesterol level in the population is different from 200 mg/dL.

Limitations of Hypothesis Testing

  • Although a useful technique, hypothesis testing does not offer a comprehensive grasp of the topic being studied. Without fully reflecting the intricacy or whole context of the phenomena, it concentrates on certain hypotheses and statistical significance.
  • The accuracy of hypothesis testing results is contingent on the quality of available data and the appropriateness of statistical methods used. Inaccurate data or poorly formulated hypotheses can lead to incorrect conclusions.
  • Relying solely on hypothesis testing may cause analysts to overlook significant patterns or relationships in the data that are not captured by the specific hypotheses being tested. This limitation underscores the importance of complimenting hypothesis testing with other analytical approaches.

Hypothesis testing stands as a cornerstone in statistical analysis, enabling data scientists to navigate uncertainties and draw credible inferences from sample data. By systematically defining null and alternative hypotheses, choosing significance levels, and leveraging statistical tests, researchers can assess the validity of their assumptions. The article also elucidates the critical distinction between Type I and Type II errors, providing a comprehensive understanding of the nuanced decision-making process inherent in hypothesis testing. The real-life example of testing a new drug’s effect on blood pressure using a paired T-test showcases the practical application of these principles, underscoring the importance of statistical rigor in data-driven decision-making.

Frequently Asked Questions (FAQs)

1. what are the 3 types of hypothesis test.

There are three types of hypothesis tests: right-tailed, left-tailed, and two-tailed. Right-tailed tests assess if a parameter is greater, left-tailed if lesser. Two-tailed tests check for non-directional differences, greater or lesser.

2.What are the 4 components of hypothesis testing?

Null Hypothesis ( [Tex]H_o [/Tex] ): No effect or difference exists. Alternative Hypothesis ( [Tex]H_1 [/Tex] ): An effect or difference exists. Significance Level ( [Tex]\alpha [/Tex] ): Risk of rejecting null hypothesis when it’s true (Type I error). Test Statistic: Numerical value representing observed evidence against null hypothesis.

3.What is hypothesis testing in ML?

Statistical method to evaluate the performance and validity of machine learning models. Tests specific hypotheses about model behavior, like whether features influence predictions or if a model generalizes well to unseen data.

4.What is the difference between Pytest and hypothesis in Python?

Pytest purposes general testing framework for Python code while Hypothesis is a Property-based testing framework for Python, focusing on generating test cases based on specified properties of the code.

Please Login to comment...

Similar reads.

  • data-science
  • Best PS5 SSDs in 2024: Top Picks for Expanding Your Storage
  • Best Nintendo Switch Controllers in 2024
  • Xbox Game Pass Ultimate: Features, Benefits, and Pricing in 2024
  • Xbox Game Pass vs. Xbox Game Pass Ultimate: Which is Right for You?
  • Full Stack Developer Roadmap [2024 Updated]

Improve your Coding Skills with Practice

 alt=

What kind of Experience do you want to share?

Stack Exchange Network

Stack Exchange network consists of 183 Q&A communities including Stack Overflow , the largest, most trusted online community for developers to learn, share their knowledge, and build their careers.

Q&A for work

Connect and share knowledge within a single location that is structured and easy to search.

What exactly is a hypothesis space in machine learning?

Whilst I understand the term conceptually, I'm struggling to understand it operationally. Could anyone help me out by providing an example?

  • machine-learning
  • terminology

Five σ's user avatar

  • $\begingroup$ A space where we can predict output by a set of some legal hypothesis (or function) and function is represented in terms of features. $\endgroup$ –  Abhishek Kumar Commented Aug 9, 2019 at 17:03

3 Answers 3

Lets say you have an unknown target function $f:X \rightarrow Y$ that you are trying to capture by learning . In order to capture the target function you have to come up with some hypotheses, or you may call it candidate models denoted by H $h_1,...,h_n$ where $h \in H$ . Here, $H$ as the set of all candidate models is called hypothesis class or hypothesis space or hypothesis set .

For more information browse Abu-Mostafa's presentaton slides: https://work.caltech.edu/textbook.html

pentanol's user avatar

  • 8 $\begingroup$ This answer conveys absolutely no information! What is the intended relationship between $f$, $h$, and $H$? What is meant by "hypothesis set"? $\endgroup$ –  whuber ♦ Commented Nov 28, 2015 at 20:50
  • 5 $\begingroup$ Please take a few minutes with our help center to learn about this site and its standards, JimBoy. $\endgroup$ –  whuber ♦ Commented Nov 28, 2015 at 20:57
  • $\begingroup$ The answer says very clear, h learns to capture target function f . H is the space where h1, h2,..hn got defined. $\endgroup$ –  Logan Commented Nov 29, 2018 at 21:47
  • $\begingroup$ @whuber I hope this is clearer $\endgroup$ –  pentanol Commented Aug 6, 2021 at 8:51
  • $\begingroup$ @pentanol You have succeeded in providing a different name for "hypothesis space," but without a definition or description of "candidate model," it doesn't seem to add any information to the post. What would be useful is information relevant to the questions that were posed, which concern "understand[ing] operationally" and a request for an example. $\endgroup$ –  whuber ♦ Commented Aug 6, 2021 at 13:55

Suppose an example with four binary features and one binary output variable. Below is a set of observations:

This set of observations can be used by a machine learning (ML) algorithm to learn a function f that is able to predict a value y for any input from the input space .

We are searching for the ground truth f(x) = y that explains the relation between x and y for all possible inputs in the correct way.

The function f has to be chosen from the hypothesis space .

To get a better idea: The input space is in the above given example $2^4$ , its the number of possible inputs. The hypothesis space is $2^{2^4}=65536$ because for each set of features of the input space two outcomes ( 0 and 1 ) are possible.

The ML algorithm helps us to find one function , sometimes also referred as hypothesis, from the relatively large hypothesis space.

  • A Few Useful Things to Know About ML

Lerner Zhang's user avatar

  • 1 $\begingroup$ Just a small note on your answer: the size of the hypothesis space is indeed 65,536, but the a more easily explained expression for it would be $2^{(2^4)}$, since, there are $2^4$ possible unique samples, and thus $2^{(2^4)}$ possible label assignments for the entire input space. $\endgroup$ –  engelen Commented Jan 10, 2018 at 9:52
  • 1 $\begingroup$ @engelen Thanks for your advice, I've edited the answer. $\endgroup$ –  So S Commented Jan 10, 2018 at 21:00
  • $\begingroup$ @SoS That one function is called classifier?? $\endgroup$ –  user125163 Commented Aug 22, 2018 at 16:26
  • 2 $\begingroup$ @Arjun Hedge: Not the one, but one function that you learned is the classifier. The classifier could be (and that's your aim) the one function. $\endgroup$ –  So S Commented Aug 22, 2018 at 16:50

The hypothesis space is very relevant to the topic of the so-called Bias-Variance Tradeoff in maximum likelihood. That's if the number of parameters in the model(hypothesis function) is too small for the model to fit the data(indicating underfitting and that the hypothesis space is too limited), the bias is high; while if the model you choose contains too many parameters than needed to fit the data the variance is high(indicating overfitting and that the hypothesis space is too expressive).

As stated in So S ' answer, if the parameters are discrete we can easily and concretely calculate how many possibilities are in the hypothesis space(or how large it is), but normally under realy life circumstances the parameters are continuous. Therefore generally the hypothesis space is uncountable.

Here is an example I borrowed and modified from the related part in the classical machine learning textbook: Pattern Recognition And Machine Learning to fit this question:

We are selecting a hypothesis function for an unknown function hidding in the training data given by a third person named CoolGuy living in an extragalactic planet. Let's say CoolGuy knows what the function is, because the data cases are provided by him and he just generated the data using the function. Let's call it(we only have the limited data and CoolGuy has both the unlimited data and the function generating them) the ground truth function and denote it by $y(x, w)$ .

enter image description here

The green curve is the $y(x,w)$ , and the little blue circles are the cases we have(they are not actually the true data cases transmitted by CoolGuy because of the it would be contaminated by some transmission noise, for example by macula or other things).

We thought that that hidden function would be very simple then we make an attempt at a linear model(make a hypothesis with a very limited space): $g_1(x, w)=w_0 + w_1 x$ with only two parameters: $w_0$ and $w_1$ , and we train the model use our data and we obtain this:

enter image description here

We can see that no matter how many data we use to fit the hypothesis it just doesn't work because it is not expressive enough.

So we try a much more expressive hypothesis: $g_9=\sum_j^9 w_j x^j $ with ten adaptive paramaters $w_0, w_1\cdots , w_9$ , and we also train the model and then we get:

enter image description here

We can see that it is just too expressive and fits all data cases. We see that a much larger hypothesis space( since $g_2$ can be expressed by $g_9$ by setting $w_2, w_3, \cdots, w_9$ as all 0 ) is more powerful than a simple hypothesis. But the generalization is also bad. That is, if we recieve more data from CoolGuy and to do reference, the trained model most likely fails in those unseen cases.

Then how large the hypothesis space is large enough for the training dataset? We can find an aswer from the textbook aforementioned:

One rough heuristic that is sometimes advocated is that the number of data points should be no less than some multiple (say 5 or 10) of the number of adaptive parameters in the model.

And you'll see from the textbook that if we try to use 4 parameters, $g_3=w_0+w_1 x + w_2 x^2 + w_3 x^3$ , the trained function is expressive enough for the underlying function $y=\sin(2\pi x)$ . It's kind a black art to find the number 3(the appropriate hypothesis space) in this case.

Then we can roughly say that the hypothesis space is the measure of how expressive you model is to fit the training data. The hypothesis that is expressive enough for the training data is the good hypothesis with an expressive hypothesis space. To test whether the hypothesis is good or bad we do the cross validation to see if it performs well in the validation data-set. If it is neither underfitting(too limited) nor overfititing(too expressive) the space is enough(according to Occam Razor a simpler one is preferable, but I digress).

  • $\begingroup$ This approach looks relevant, but your explanation does not agree with that on p. 5 of your first reference: "A function $h:X\to\{0,1\}$ is called [an] hypothesis. A set $H$ of hypotheses among which the approximation function $y$ is searched is called [the] hypothesis space." (I would agree the slide is confusing, because its explanation implicitly requires that $C=\{0,1\}$, whereas that is generically labeled "classes" in the diagram. But let's not pass along that confusion: let's rectify it.) $\endgroup$ –  whuber ♦ Commented Sep 24, 2016 at 15:33
  • 1 $\begingroup$ @whuber I updated my answer just now more than two years later after I have learned more knowledge on the topic. Please help check if I can rectify it in a better way. Thanks. $\endgroup$ –  Lerner Zhang Commented Feb 5, 2019 at 11:41

Not the answer you're looking for? Browse other questions tagged machine-learning terminology definition or ask your own question .

  • Featured on Meta
  • Site maintenance - Mon, Sept 16 2024, 21:00 UTC to Tue, Sept 17 2024, 2:00...
  • User activation: Learnings and opportunities
  • Join Stack Overflow’s CEO and me for the first Stack IRL Community Event in...

Hot Network Questions

  • Does the word vaishnava appear even once in Srimad Bhagavatam?
  • Why is resonance such a widespread phenomenon?
  • Problem with the hinge mechanism of my folding bike (Smart Truck 300, made ~2016)
  • Why would the GPL be viral, while EUPL isn't, according to the EUPL authors?
  • Is it a correct rendering of Acts 1,24 when the New World Translation puts in „Jehovah“ instead of Lord?
  • Why are some Cloudflare challenges CPU intensive?
  • Example of two dinatural transformations between finite categories that do not compose
  • How to prove that the Greek cross tiles the plane?
  • What film is it where the antagonist uses an expandable triple knife?
  • How can I stop the noise from updating its position when I adjust the spline?
  • Connections vertically and horizontally
  • why the eigen values of a random matrix are not the same?
  • What was the main implementation programming language of old 16-bit Windows versions (Windows 1 - Windows 3.11)?
  • Does any row of Pascal's triangle contain a Pythagorean triple?
  • Copyright Fair Use: Is using the phrase "Courtesy of" legally acceptable when no permission has been given?
  • A journal has published an AI-generated article under my name. What to do?
  • PCB layout guidelines Magnetic Sensor
  • What is the rationale behind 32333 "Technic Pin Connector Block 1 x 5 x 3"?
  • Is there mathematical significance to the LaGuardia floor tiles?
  • How can I analyze the anatomy of a humanoid species to create sounds for their language?
  • Interpretation of Amulet from Moroccan Rabbi
  • Tire schrader core replacement without deflation
  • How would platypus evolve some sort of digestive acid?
  • How did NASA figure out when and where the Apollo capsule would touch down on the ocean?

use of hypothesis in machine learning

Machine Learning as a Tool for Hypothesis Generation

While hypothesis testing is a highly formalized activity, hypothesis generation remains largely informal. We propose a systematic procedure to generate novel hypotheses about human behavior, which uses the capacity of machine learning algorithms to notice patterns people might not. We illustrate the procedure with a concrete application: judge decisions about who to jail. We begin with a striking fact: The defendant’s face alone matters greatly for the judge’s jailing decision. In fact, an algorithm given only the pixels in the defendant’s mugshot accounts for up to half of the predictable variation. We develop a procedure that allows human subjects to interact with this black-box algorithm to produce hypotheses about what in the face influences judge decisions. The procedure generates hypotheses that are both interpretable and novel: They are not explained by demographics (e.g. race) or existing psychology research; nor are they already known (even if tacitly) to people or even experts. Though these results are specific, our procedure is general. It provides a way to produce novel, interpretable hypotheses from any high-dimensional dataset (e.g. cell phones, satellites, online behavior, news headlines, corporate filings, and high-frequency time series). A central tenet of our paper is that hypothesis generation is in and of itself a valuable activity, and hope this encourages future work in this largely “pre-scientific” stage of science.

This is a revised version of Chicago Booth working paper 22-15 “Algorithmic Behavioral Science: Machine Learning as a Tool for Scientific Discovery.” We gratefully acknowledge support from the Alfred P. Sloan Foundation, Emmanuel Roman, and the Center for Applied Artificial Intelligence at the University of Chicago. For valuable comments we thank Andrei Shliefer, Larry Katz and five anonymous referees, as well as Marianne Bertrand, Jesse Bruhn, Steven Durlauf, Joel Ferguson, Emma Harrington, Supreet Kaur, Matteo Magnaricotte, Dev Patel, Betsy Levy Paluck, Roberto Rocha, Evan Rose, Suproteem Sarkar, Josh Schwartzstein, Nick Swanson, Nadav Tadelis, Richard Thaler, Alex Todorov, Jenny Wang and Heather Yang, as well as seminar participants at Bocconi, Brown, Columbia, ETH Zurich, Harvard, MIT, Stanford, the University of California Berkeley, the University of Chicago, the University of Pennsylvania, the 2022 Behavioral Economics Annual Meetings and the 2022 NBER summer institute. For invaluable assistance with the data and analysis we thank Cecilia Cook, Logan Crowl, Arshia Elyaderani, and especially Jonas Knecht and James Ross. This research was reviewed by the University of Chicago Social and Behavioral Sciences Institutional Review Board (IRB20-0917) and deemed exempt because the project relies on secondary analysis of public data sources. All opinions and any errors are of course our own. The views expressed herein are those of the authors and do not necessarily reflect the views of the National Bureau of Economic Research.

MARC RIS BibTeΧ

Download Citation Data

Published Versions

Jens Ludwig & Sendhil Mullainathan, 2024. " Machine Learning as a Tool for Hypothesis Generation, " The Quarterly Journal of Economics, vol 139(2), pages 751-827.

Working Groups

Conferences, more from nber.

In addition to working papers , the NBER disseminates affiliates’ latest findings through a range of free periodicals — the NBER Reporter , the NBER Digest , the Bulletin on Retirement and Disability , the Bulletin on Health , and the Bulletin on Entrepreneurship  — as well as online conference reports , video lectures , and interviews .

2024, 16th Annual Feldstein Lecture, Cecilia E. Rouse," Lessons for Economists from the Pandemic" cover slide

This week: the arXiv Accessibility Forum

Help | Advanced Search

Computer Science > Information Theory

Title: statistically valid information bottleneck via multiple hypothesis testing.

Abstract: The information bottleneck (IB) problem is a widely studied framework in machine learning for extracting compressed features that are informative for downstream tasks. However, current approaches to solving the IB problem rely on a heuristic tuning of hyperparameters, offering no guarantees that the learned features satisfy information-theoretic constraints. In this work, we introduce a statistically valid solution to this problem, referred to as IB via multiple hypothesis testing (IB-MHT), which ensures that the learned features meet the IB constraints with high probability, regardless of the size of the available dataset. The proposed methodology builds on Pareto testing and learn-then-test (LTT), and it wraps around existing IB solvers to provide statistical guarantees on the IB constraints. We demonstrate the performance of IB-MHT on classical and deterministic IB formulations, validating the effectiveness of IB-MHT in outperforming conventional methods in terms of statistical robustness and reliability.
Subjects: Information Theory (cs.IT); Machine Learning (cs.LG)
Cite as: [cs.IT]
  (or [cs.IT] for this version)
  Focus to learn more arXiv-issued DOI via DataCite (pending registration)

Submission history

Access paper:.

  • HTML (experimental)
  • Other Formats

license icon

References & Citations

  • Google Scholar
  • Semantic Scholar

BibTeX formatted citation

BibSonomy logo

Bibliographic and Citation Tools

Code, data and media associated with this article, recommenders and search tools.

  • Institution

arXivLabs: experimental projects with community collaborators

arXivLabs is a framework that allows collaborators to develop and share new arXiv features directly on our website.

Both individuals and organizations that work with arXivLabs have embraced and accepted our values of openness, community, excellence, and user data privacy. arXiv is committed to these values and only works with partners that adhere to them.

Have an idea for a project that will add value for arXiv's community? Learn more about arXivLabs .

IMAGES

  1. Hypothesis in Machine Learning

    use of hypothesis in machine learning

  2. What is Hypothesis Testing?

    use of hypothesis in machine learning

  3. Hypothesis Testing In Machine Learning While Using Python- Tutorial

    use of hypothesis in machine learning

  4. Hypothesis in Machine Learning. Written by: Preeti Yadav(201550105, GLA

    use of hypothesis in machine learning

  5. Hypothesis in Machine Learning

    use of hypothesis in machine learning

  6. Hypothesis in Machine Learning

    use of hypothesis in machine learning

VIDEO

  1. Decision Tree :: Decision Tree Hypothesis @ Machine Learning Techniques (機器學習技法)

  2. Radial Basis Function Network :: RBF Network Hypothesis @ Machine Learning Techniques (機器學習技法)

  3. Hypothesis Representation Stanford University Coursera

  4. Concept of Hypothesis

  5. Unit-1 Machine Learning

  6. Neural Network :: Neural Network Hypothesis @ Machine Learning Techniques (機器學習技法)

COMMENTS

  1. What is a Hypothesis in Machine Learning?

    Hypothesis in Machine Learning: Candidate model that approximates a target function for mapping examples of inputs to outputs. We can see that a hypothesis in machine learning draws upon the definition of a hypothesis more broadly in science. Just like a hypothesis in science is an explanation that covers available evidence, is falsifiable and ...

  2. Hypothesis in Machine Learning

    A hypothesis is a function that best describes the target in supervised machine learning. The hypothesis that an algorithm would come up depends upon the data and also depends upon the restrictions and bias that we have imposed on the data. The Hypothesis can be calculated as: y = mx + b y =mx+b. Where, y = range. m = slope of the lines.

  3. Hypothesis in Machine Learning

    The hypothesis is one of the commonly used concepts of statistics in Machine Learning. It is specifically used in Supervised Machine learning, where an ML model learns a function that best maps the input to corresponding outputs with the help of an available dataset. In supervised learning techniques, the main aim is to determine the possible ...

  4. Everything you need to know about Hypothesis Testing in Machine Learning

    The null hypothesis represented as H₀ is the initial claim that is based on the prevailing belief about the population. The alternate hypothesis represented as H₁ is the challenge to the null hypothesis. It is the claim which we would like to prove as True. One of the main points which we should consider while formulating the null and alternative hypothesis is that the null hypothesis ...

  5. Hypothesis Testing

    Foundations Of Machine Learning (Free) Python Programming(Free) Numpy For Data Science(Free) Pandas For Data Science(Free) ... ($α$) 0.05: the results are not statistically significant, and they don't reject the null hypothesis, remaining unsure if the drug has a genuine effect. 4. Example in python. For simplicity, let's say we're using ...

  6. Machine Learning: The Basics

    A learning rate or step-size parameter used by gradient-based methods. h() A hypothesis map that reads in features x of a data point and delivers a prediction ^y= h(x) for its label y. H A hypothesis space or model used by a ML method. The hypothesis space consists of di erent hypothesis maps h: X!Ybetween which the ML method has to choose. 8

  7. Hypothesis Testing in Machine Learning

    The steps involved in the hypothesis testing are as follow: Assume a null hypothesis, usually in machine learning algorithms we consider that there is no anomaly between the target and independent variable. Collect a sample. Calculate test statistics. Decide either to accept or reject the null hypothesis.

  8. A Gentle Introduction to Statistical Hypothesis Testing

    A statistical hypothesis test may return a value called p or the p-value. This is a quantity that we can use to interpret or quantify the result of the test and either reject or fail to reject the null hypothesis. This is done by comparing the p-value to a threshold value chosen beforehand called the significance level.

  9. Best Guesses: Understanding The Hypothesis in Machine Learning

    In machine learning, the term 'hypothesis' can refer to two things. First, it can refer to the hypothesis space, the set of all possible training examples that could be used to predict or answer a new instance. Second, it can refer to the traditional null and alternative hypotheses from statistics. Since machine learning works so closely ...

  10. Hypothesis in Machine Learning: Comprehensive Overview(2021)

    This is identified with the bias-variance trade-off. A hypothesis function in machine learning is best describes the target. The hypothesis that an algorithm would concoct relies on the data and relies on the bias and restrictions that we have forced on the data. The hypothesis formula in machine learning: y= mx b.

  11. Hypothesis

    Null Hypothesis. The Null Hypothesis is position that there is no relationship between two measured groups. An example is the development of a new pharmaceutical drug, where the Null Hypothesis is that the drug is considered not effective. The Null Hypothesis is often referred to as H0 (H zero).

  12. Machine Learning

    In machine learning, a hypothesis is a proposed explanation or solution for a problem. It is a tentative assumption or idea that can be tested and validated using data. In supervised learning, the hypothesis is the model that the algorithm is trained on to make predictions on unseen data. The hypothesis is generally expressed as a function that ...

  13. Evaluating Hypotheses in Machine Learning: A Comprehensive Guide

    In machine learning, a hypothesis is a statement that proposes a possible explanation for a phenomenon or a problem. It is a conjecture that is made about a population parameter, and it is used as a basis for further investigation. In the context of machine learning, hypotheses are used to define the problem that we are trying to solve.

  14. Hypothesis Test for Comparing Machine Learning Algorithms

    In this tutorial, you will discover how to use statistical hypothesis tests for comparing machine learning algorithms. After completing this tutorial, you will know: Performing model selection based on the mean model performance can be misleading. The five repeats of two-fold cross-validation with a modified Student's t-Test is a good ...

  15. Hypothesis testing in Machine learning using Python

    Now Let's see some of widely used hypothesis testing type :-T Test ( Student T test) Z Test; ANOVA Test; Chi-Square Test; T- Test :- A t-test is a type of inferential statistic which is used to determine if there is a significant difference between the means of two groups which may be related in certain features.It is mostly used when the data sets, like the set of data recorded as outcome ...

  16. What's a Hypothesis Space?

    Our goal is to find a model that classifies objects as positive or negative. Applying Logistic Regression, we can get the models of the form: (1) which estimate the probability that the object at hand is positive. Each such model is called a hypothesis, while the set of all the hypotheses an algorithm can learn is known as its hypothesis space ...

  17. Understanding Hypothesis Testing

    The concept of a hypothesis is fundamental in Machine Learning and data science endeavours. In the realm of machine learning, a hypothesis serves as an initial assumption made by data scientists and ML professionals when attempting to address a problem. Machine learning involves conducting experiments based on past experiences, and these hypotheses

  18. Introduction of Hypothesis in Statistics and Machine Learning

    Here,we will study the difference between a hypothesis in science, in statistics, and in machine learning. Table of content:-What is Hypothesis? Hypothesis in Statistics. Hypothesis in Machine ...

  19. What is Hypothesis in Machine Learning? How to Form a Hypothesis?

    The hypothesis is a crucial aspect of Machine Learning and Data Science. It is present in all the domains of analytics and is the deciding factor of whether a change should be introduced or not. Be it pharma, software, sales, etc. A Hypothesis covers the complete training dataset to check the performance of the models from the Hypothesis space.

  20. 17 Statistical Hypothesis Tests in Python (Cheat Sheet)

    In this post, you will discover a cheat sheet for the most popular statistical hypothesis tests for a machine learning project with examples using the Python API. Each statistical test is presented in a consistent way, including: The name of the test. What the test is checking. The key assumptions of the test. How the test result is interpreted.

  21. Evaluate ML Classifiers Performance using Hypothesis testing

    Types of statistical hypothesis testings. Examining machine learning models via statistical significance tests requires some expectations that will influence the statistical tests used. The most robust way to such comparisons is called paired designs, which compare both models (or algorithms) performance on the same data. That way, both models ...

  22. What exactly is a hypothesis space in machine learning?

    Here is an example I borrowed and modified from the related part in the classical machine learning textbook: Pattern Recognition And Machine Learning to fit this question: We are selecting a hypothesis function for an unknown function hidding in the training data given by a third person named CoolGuy living in an extragalactic planet.

  23. Machine Learning as a Tool for Hypothesis Generation

    Jens Ludwig & Sendhil Mullainathan, 2024. "Machine Learning as a Tool for Hypothesis Generation," The Quarterly Journal of Economics, vol 139 (2), pages 751-827. Founded in 1920, the NBER is a private, non-profit, non-partisan organization dedicated to conducting economic research and to disseminating research findings among academics, public ...

  24. Statistically Valid Information Bottleneck via Multiple Hypothesis Testing

    The information bottleneck (IB) problem is a widely studied framework in machine learning for extracting compressed features that are informative for downstream tasks. However, current approaches to solving the IB problem rely on a heuristic tuning of hyperparameters, offering no guarantees that the learned features satisfy information-theoretic constraints. In this work, we introduce a ...

  25. Filling the Gaps: A Comparative Guide to Imputation Techniques in

    Currently, I channel my passion for data science, machine learning, and AI as a Mentor at the New York City Data Science Academy. I value the opportunity to ignite curiosity and share knowledge, whether through Live Learning sessions or in-depth 1-on-1 interactions. With a foundation in finance/entrepreneurship and my current immersion in the ...